Difference between revisions of "2003 AMC 10A Problems/Problem 16"

(added problem and solution)
 
(Solution 2 (Patterns))
 
(5 intermediate revisions by 5 users not shown)
Line 4: Line 4:
 
<math> \mathrm{(A) \ } 1\qquad \mathrm{(B) \ } 3\qquad \mathrm{(C) \ } 7\qquad \mathrm{(D) \ } 8\qquad \mathrm{(E) \ } 9 </math>
 
<math> \mathrm{(A) \ } 1\qquad \mathrm{(B) \ } 3\qquad \mathrm{(C) \ } 7\qquad \mathrm{(D) \ } 8\qquad \mathrm{(E) \ } 9 </math>
  
== Solution ==
+
== Solution 1==
 
<math>13^{2003}\equiv 3^{2003}\pmod{10}</math>
 
<math>13^{2003}\equiv 3^{2003}\pmod{10}</math>
  
Line 11: Line 11:
 
<math>3^{2003}=(3^{4})^{500}\cdot3^{3}\equiv1^{500}\cdot27\equiv7\pmod{10}</math>
 
<math>3^{2003}=(3^{4})^{500}\cdot3^{3}\equiv1^{500}\cdot27\equiv7\pmod{10}</math>
  
Therefore, the units digit is <math>7 \Rightarrow C</math>
+
Therefore, the units digit is <math>7 \Rightarrow\boxed{\mathrm{(C)}\ 7}</math>
 +
 
 +
== Solution 2 (Patterns) ==
 +
Since we are looking for the units digit of <math>13^{2003}</math>, we only have to focus on the units digit of the base (13) as none of the other digits of the base affect the units digit of the resulting value.
 +
 
 +
By testing the first few values or through previous knowledge, you might see that the units digit of exponents with base 3 follow this pattern:
 +
<cmath>3^1=3</cmath>
 +
<cmath>3^2=9</cmath>
 +
<cmath>3^3=27</cmath>
 +
<cmath>3^4=81,</cmath>
 +
giving us the rotation <math>3-9-7-1.</math>
 +
 
 +
As this cycle resets every time the index increases by 4, we know that this cycle ends on 2000, and starts once again on 2001. As our expression is raised to the power of 2003, we know that the units digit of our expression must end with the third term of our pattern: <math>7</math>.
 +
 
 +
Therefore, the units digit of our expression is <math>7 \Rightarrow\boxed{\mathrm{(C)}\ 7}</math>
 +
 
 +
~ JinhoK
  
 
== See Also ==
 
== See Also ==
*[[2003 AMC 10A Problems]]
+
{{AMC10 box|year=2003|ab=A|num-b=15|num-a=17}}
*[[2003 AMC 10A Problems/Problem 15|Previous Problem]]
 
*[[2003 AMC 10A Problems/Problem 17|Next Problem]]
 
  
 
[[Category:Introductory Number Theory Problems]]
 
[[Category:Introductory Number Theory Problems]]
 +
{{MAA Notice}}

Latest revision as of 15:52, 19 August 2023

Problem

What is the units digit of $13^{2003}$?

$\mathrm{(A) \ } 1\qquad \mathrm{(B) \ } 3\qquad \mathrm{(C) \ } 7\qquad \mathrm{(D) \ } 8\qquad \mathrm{(E) \ } 9$

Solution 1

$13^{2003}\equiv 3^{2003}\pmod{10}$

Since $3^4=81\equiv1\pmod{10}$:

$3^{2003}=(3^{4})^{500}\cdot3^{3}\equiv1^{500}\cdot27\equiv7\pmod{10}$

Therefore, the units digit is $7 \Rightarrow\boxed{\mathrm{(C)}\ 7}$

Solution 2 (Patterns)

Since we are looking for the units digit of $13^{2003}$, we only have to focus on the units digit of the base (13) as none of the other digits of the base affect the units digit of the resulting value.

By testing the first few values or through previous knowledge, you might see that the units digit of exponents with base 3 follow this pattern: \[3^1=3\] \[3^2=9\] \[3^3=27\] \[3^4=81,\] giving us the rotation $3-9-7-1.$

As this cycle resets every time the index increases by 4, we know that this cycle ends on 2000, and starts once again on 2001. As our expression is raised to the power of 2003, we know that the units digit of our expression must end with the third term of our pattern: $7$.

Therefore, the units digit of our expression is $7 \Rightarrow\boxed{\mathrm{(C)}\ 7}$

~ JinhoK

See Also

2003 AMC 10A (ProblemsAnswer KeyResources)
Preceded by
Problem 15
Followed by
Problem 17
1 2 3 4 5 6 7 8 9 10 11 12 13 14 15 16 17 18 19 20 21 22 23 24 25
All AMC 10 Problems and Solutions

The problems on this page are copyrighted by the Mathematical Association of America's American Mathematics Competitions. AMC logo.png